Đến nội dung

Primary

Primary

Đăng ký: 19-11-2012
Offline Đăng nhập: 20-08-2021 - 16:03
***--

#382665 Tính R1-R2+R3-R4

Gửi bởi Primary trong 01-01-2013 - 20:08

phải đề OLP 30 tháng 4 ko vậy bạn

Đúng vậy, thấy bài này tính hay hay post lên
Cái này không cần dùng tích phân cũng được


#382349 $\sum \frac{\sqrt{b+c-a}}{\sqrt{b}+\sqrt{c}-...

Gửi bởi Primary trong 31-12-2012 - 20:14

Cho a, b, c là 3 cạnh của tam giác. Chứng minh rằng:$$\sum \frac{\sqrt{b+c-a}}{\sqrt{b}+\sqrt{c}-\sqrt{a}}\leq 3$$


#382315 Giải bất phương trình: $\frac{x^2-x-6}{(x^2-9)(x+1)...

Gửi bởi Primary trong 31-12-2012 - 18:26

Giải:

$\frac{x^2-x-6}{(x^2-9)(x+1)}\leq 0$

$\Leftrightarrow \frac{(x-3)(x+2)}{(x-3)(x+3)(x+1)}\leq 0$ (1)

$x-3=0 \Leftrightarrow x=3$
$x+2=0 \Leftrightarrow x=-2$
$x-3=0 \Leftrightarrow x=3$
$x+3=0 \Leftrightarrow x=-3$
$x+1=0 \Leftrightarrow x=-1$

Bảng xét dấu:
.............
..............


Cho em hỏi tại sao ở chỗ (1) ta không triệt tiêu cái (x-3) luôn vậy?

Nhưng em nghĩ là dù x lớn hơn hay nhỏ hơn 3 thì trên chia dưới cũng bằng 1 thôi. Không hiểu thiệt.

Vì ta chưa biết $x>3$ hay không. Mình nghĩ thế ^^. Không biết đúng không!

Vấn đề thực chất là x=3 thì đa VP vô nghĩa
Nên có thể triệt tiêu sau khi xét x khác 3


#382195 Chứng minh rằng tồn tại $2$ điểm có khoảng cách nhỏ hơn $2$.

Gửi bởi Primary trong 31-12-2012 - 10:06

Bài toán: Cho đường tròn $\left ( O;2,5 \right )$. Chứng minh rằng trong $10$ điểm nằm bên trong đường tròn thì tồn tại $2$ điểm có khoảng cách nhỏ hơn $2$.

Em chỉ biết thế này:
Gọi $d(X_i,X_{i+1})$ là khoảng cách giữa 2 điểm $X_i,X_{i+1}$
Chu vi đường tròn $C=2.2,5\pi =5\pi$ $\Rightarrow$ khoảng cách trung bình giữa 2 điểm $X_i,X_{i+1}$ là $d(X_i,X_{i+1})=\frac{\pi }{2}<2$
Nếu khoảng cách giữa 2 điểm $X_i,X_{i+1}$ là $d(X_i,X_{i+1})$>2 suy ra có ít nhất 2 điểm trong 10 điểm có khoảng cách $d(X_k,X_{k+1})$<2 với $k\neq i$
Do đó ta có điều phải chứng minh.


#381313 ÔN THI ĐẠI HỌC 2013

Gửi bởi Primary trong 28-12-2012 - 20:25

Mình xin góp 1 bài:
Bài 8: Tìm GTNN của $\sum (\frac{a}{b-c})^2$, với $a\neq b\neq c$


#381309 ÔN THI ĐẠI HỌC 2013

Gửi bởi Primary trong 28-12-2012 - 20:17

Bài toán 7: Cho $x, y, z \geq 0$, tìm min của $P$ thỏa mãn:
$$P=(\sum_{sym} {xy})(\sum_{sym} \frac{1}{x^2+y^2})$$

Theo bất đẳng thức Cauchy-Schwarz:
$P\geq (\sum xy)(\sum \frac{1}{2(x+y)^2})\Leftrightarrow 2P\geq (xy+yz+zx)[\frac{1}{(x+y)^2}+\frac{1}{(y+z)^2}+\frac{1}{(z+x)^2}]$
Vế phải chính là bất đẳng thức Iran năm 1996: https://phudinhgioihan.wordpress.com/
Đến đây $2P\geq \frac{9}{4}\Leftrightarrow P\geq \frac{9}{8}$
Dấu "=" xảy ra $\Leftrightarrow x=y=z>0$


#380321 Tìm GTNN và GTLN của hàm số $y=x^4-4x^3+2x^2+4x+3 (x\in \left...

Gửi bởi Primary trong 25-12-2012 - 17:15

Thế $x=-1$ thì sao?Max $t=3$ mà.?

Mình cũng không rõ nữa cứ thế giá trị x =-1 hoặc x=2 vào là được vì x thuộc [-1;2], cái này thầy cô không ai dạy hết


#379719 $x^{2}-2y^{2}=1$

Gửi bởi Primary trong 23-12-2012 - 08:33

  • Giải phương trình nghiệm nguyên dương: Hình đã gửi.
Biểu diễn liên phân số của Hình đã gửi là:
Hình đã gửi,
biểu diễn này có chu kì r=1 lẻ, do đó nghiệm của phương trình là (Hình đã gửi) với n có dạng Hình đã gửi, các giản phân ở vị trí lẻ.
Dãy giản phân của Hình đã gửi: Hình đã gửi. Chú ý dãy số trên được bắt đầu với số thứ tự bằng 0.
Lấy các phân số ở vị trí lẻ ta được nghiệm nguyên dương của phương trình Hình đã gửi là: (3,2) (17,12), (99,70), (577,408), (3363,2378), ... và tất nhiên cả nghiệm tầm thường là (1,0).
Đây là phương trình Pell như bạn nói


#379607 Ba cạnh tam giác tỉ lệ như thế nào nếu

Gửi bởi Primary trong 22-12-2012 - 19:04

Gọi các chiều cao tương ứng tỉ lệ với $3:4:5$ là $h_1;h_2;h_3$
Ta có:$\dfrac{h_1}{3}=\dfrac{h_2}{4}=\dfrac{h_3}{5}$
Gọi các cạnh tương ứng với các đường cao là $a,b,c$,vá giả sử tỉ lệ theo$x:y:z$
Ta có $\dfrac{a}{x}=\dfrac{b}{y}=\dfrac{c}{z}$
Nhân về theo vế,ta được:
$\dfrac{ah_1}{3x}=\dfrac{bh_2}{4x}=\dfrac{ch_1}{5x}$
Vì $ah_1=bh_2=ch_3$(cùng bằng $2S_{\Delta}$)
Như vậy thì $3x=4y=5z$
Để $x:y:z$ tối giản thì $3x=4y=5z=LCM(3;4;5)=60$
Từ đây suy ra $x:y:z=20:15:12$

Hình như $h_a+h_b,h_b+h_c,h_c+h_a$ tỉ lệ với 3,4,5 thì phải :huh:


#379513 Cho hàm số: y = x4 + 2mx2 + 9. Tìm m để hàm số cắt trục hoành tại 4 điểm phâ...

Gửi bởi Primary trong 22-12-2012 - 13:10

Cho hàm số: y = x4 + 2mx2 + 9
Tìm m để hàm số cắt trục hoành tại 4 điểm phân biệt

Đặt $t=x^2,t\geq 0$$\Rightarrow y=t^2+2mt+9$
Hàm số y cắt trục hoành tại 4 điểm phân biệt $\Leftrightarrow t^2+2mt+9=0$ có 2 nghiệm dương phân biệt $\Leftrightarrow \Delta '=m^2-36> 0\Leftrightarrow m> 6\vee m< -6$


#379458 Tìm GTNN của $P=(a-b+1)^2+(b-4)^2+(2c-d+7)^2+(d+3)^2+(a-4b+3c-6d+9)^2$

Gửi bởi Primary trong 22-12-2012 - 07:51

Bài toán [Tham Lang]
Cho các số thực $a,b,c,d$. Tìm GTNN của biểu thức :
$$P=(a-b+1)^2+(b-4)^2+(2c-d+7)^2+(d+3)^2+(a-4b+3c-6d+9)^2$$

Em có cách này:
$$P=(a-b+1)^2+(b-4)^2+(2c-d+7)^2+(d+3)^2+(a-4b+3c-6d+10-1)^2$$$\geq (-1)^2\Leftrightarrow P\geq 1$
$\Rightarrow MinP=1$ khi $a=3;b=4;c=-5;d=-3$
Hình như bài toán này không mang đến 1 lời giải đẹp :icon6:


#379336 $x^2+3xz+z^2=1 \\ 3y^2+3yz+z^2=4 \\ x^2-xy+y^2=m$

Gửi bởi Primary trong 21-12-2012 - 19:47

Tìm các giá trị của tham số $m$ sao cho hệ phương trình sau có nghiệm:
$$\left\{\begin{matrix} x^2+3xz+z^2=1 (1)\\ 3y^2+3yz+z^2=4 (2) \\ x^2-xy+y^2=m (3)\end{matrix}\right.$$

Xét x=y=z=0 không phải là nghiệm của hệ
*Nếu x=0 hệ trở thành $\left\{\begin{matrix}z^2=1 & & \\ 3y^2+3yz+z^2=4 & & \\ y^2=m & & \end{matrix}\right.$ $\Rightarrow m=\frac{3\pm \sqrt{5}}{2}$
*Nếu y=0 hệ trở thành $\left\{\begin{matrix}x^2+3xz+z^2=1 & & & \\ z^2=4 & & & \\ x^2=m & & & \end{matrix}\right.$ $\Rightarrow m=15\pm 6\sqrt{6}$
*Nếu z=0 hệ trở thành $\left\{\begin{matrix}x^2=1 & & & \\ 3y^2=4 & & & \\ x^2-xy+y^2=m & & & \end{matrix}\right.$ $\Rightarrow m=\frac{7\pm 2\sqrt{3}}{4}$
*Nếu x,y,z khác 0
Đặt $x=ay,x=bz\Rightarrow x=ay=bz$ $(a,b\neq 0)$
$\Rightarrow y=\frac{bz}{a},x^2=axy$ hay $y^2=\frac{b^2z^2}{a^2},xy=\frac{x^2}{a}$
$(1)\Leftrightarrow bz^2+3bz.\frac{bz}{a}+z^2=1\Leftrightarrow z^2=\frac{a}{ab^2+a+3b^2}$
$\Rightarrow y^2=\frac{b^2}{a(ab^2+a+3b^2)},x^2=\frac{a^2b^2}{ab^2+a+3b^2}$
$(2)\Leftrightarrow \frac{3b^2}{a(ab^2+a+3b^2)}+\frac{3b.a}{a(ab^2+a+3b^2)}+\frac{a}{ab^2+a+3b^2}=4$ $\Leftrightarrow 3b^2+3ab-4a^2b^2-12ab^2-3a^2=0$ (*)
Ta có $(1)\Leftrightarrow \frac{a^2b^2}{ab^2+a+3b^2}+\frac{a}{ab62+a+3b^2}+\frac{3ab^2}{ab^2+a+3b^2}=1\Leftrightarrow (ab^2-b^2)(a+3)=0\Leftrightarrow a=1\vee a=-3,(b\neq 0)$
Với a=1 thay vào (*) ta được $-13b^2+3b-3=0$ (Vô nghiệm)
Với a=-3 thay vào (*) ta được $b^2-3b-9=0\Leftrightarrow b=\frac{3\pm 3\sqrt{5}}{2}$
Khi đó $x=-3y$ và thay vào (1) ta được $z^2=1\Leftrightarrow z=\pm 1$
Nếu $b=\frac{3+3\sqrt{5}}{2}$ thì:
* z=1$\Rightarrow y=-\frac{1+\sqrt{5}}{2}\Rightarrow x=\frac{3+3\sqrt{5}}{2}$
$(3)\Leftrightarrow m=\frac{39+13\sqrt{5}}{2}$
* z=-1$\Rightarrow y=\frac{1+\sqrt{5}}{2}\Rightarrow x=-\frac{3+3\sqrt{5}}{2}$
$(3)\Leftrightarrow m=\frac{39+13\sqrt{5}}{2}$
Nếu $b=\frac{3-3\sqrt{5}}{2}$ thì
* z=1 $\Rightarrow y=\frac{-1+\sqrt{5}}{2}\Rightarrow x=\frac{3-3\sqrt{5}}{2}$
$(3)\Leftrightarrow \frac{39-13\sqrt{5}}{2}$
* z=-1 $\Rightarrow y=\frac{1-\sqrt{5}}{2}\Rightarrow x=\frac{-3+3\sqrt{5}}{2}$
$(3)\Leftrightarrow m=\frac{39-13\sqrt{5}}{2}$
Thử lại m thỏa yêu cầu bài toán
Vậy $m\in \left \{ \frac{39\pm 13\sqrt{5}}{2};\frac{3\pm \sqrt{5}}{2};15\pm 6\sqrt{6};\frac{7\pm 2\sqrt{3}}{4} \right \}$ thì hệ có nghiệm


#379054 Tìm số thực $x$ thỏa mãn điều kiện sau : $\sum \left...

Gửi bởi Primary trong 20-12-2012 - 14:27

Thầy thanh nói đúng đó, cái này hiển nhiên là sai khi ta chọn khéo phần lẻ của $x$
$x=[x]+\{x\}$
Nên $[nx]=[n[x]+n\{x\}]=n[x]+[n\{x\}]$
Đến đây lựa chọn cẩn thận ta sẽ chọn được $\{x\}\geq \dfrac{2}{n}$ (với $n\geq 3$) mà vẫn đảm bảo được $\{x\}<1$ khi ấy $[n\{x\}]\geq 2$ nên BDT của bạn đã bị sai :)

Lúc đầu nhìn vào thử mọi số thấy đúng nhưng chứng minh không được và thử như nguyenta98 thì kết quả lại sai. Vậy chẳng lẽ chỉ có duy nhất 1 cách hay sao ???


#379052 $\sum \dfrac{x}{y+2z} \ge 1$

Gửi bởi Primary trong 20-12-2012 - 14:15

Chứng minh với mọi số $a,b,c \in \mathbb{R^{+}}$,ta luôn có:
$\dfrac{x}{y+2z}+\dfrac{y}{z+2x}+\dfrac{z}{x+2y} \ge 1$
________
Không khó đâu :D

Theo bất đẳng thức Cauchy-Schwarz
$\sum \frac{x}{y+2z}=\sum \frac{x^2}{xy+2xz}\geq \frac{(x+y+z)^2}{3(xy+yz+xz)}\geq \frac{(x+y+z)^2}{3(x^2+y^2+z^2)}\geq 1$ ??


#379050 Tìm số thực $x$ thỏa mãn điều kiện sau : $\sum \left...

Gửi bởi Primary trong 20-12-2012 - 13:51

@: Primary
Em chứng minh cái "dễ thấy" này đi.
$n\lfloor x\rfloor\le\lfloor nx\rfloor\le n\lfloor x\rfloor+1,\;\forall n\in\mathbb N^*$

*Với mọi số thực x, y thì:
$[x+y]=[[x]+[y]+\left \{ x \right \}+\left \{ y \right \}]=[x]+[y]+[\left \{ x \right \}+\left \{ y \right \}]$
Vì $0\leq \left \{ x \right \},\left \{ y \right \}<1$ nên
$\Rightarrow [x]+[y]\leq [x+y]\leq [x+y]+1$ (*)
1) Chứng minh: $n[x]\leq [nx]$
Áp dụng (*) ta có : $n[x]=[x]+...+[x]\leq [x+x]+[x]+...+[x]\leq ...\leq [x+...+x]=n[x]\Leftrightarrow n[x]\leq [nx]$
Do đó (1) được chứng minh
2) Chứng minh $[nx]\leq n[x]+1$
*Nếu $n=2k, k\in \mathbb{N}$:
$[nx]=[kx+kx]\leq [kx]+[kx]+1=2[kx]+1\leq [2kx]+1\Leftrightarrow [nx]\leq n[x]+1$
*Nếu $n=2k+1,k\in \mathbb{N}$: (phần này cứng như đá)